ΘΕΜΑΤΑ JBMO 2015

Συντονιστές: cretanman, ΔΗΜΗΤΡΗΣ ΙΩΑΝΝΟΥ, socrates

Άβαταρ μέλους
Demetres
Γενικός Συντονιστής
Δημοσιεύσεις: 8989
Εγγραφή: Δευ Ιαν 19, 2009 5:16 pm
Τοποθεσία: Λεμεσός/Πύλα
Επικοινωνία:

ΘΕΜΑΤΑ JBMO 2015

#1

Μη αναγνωσμένη δημοσίευση από Demetres » Παρ Ιουν 26, 2015 3:48 pm

Ας αφήσουμε το νήμα εδώ για τις ευχές και τα (αναμενόμενα) συγχαρητήρια και ας συζητήσουμε εδώ τα θέματα της βαλκανιάδας.

Ευχαριστώ το μέλος μας Σωτήρη Λοϊζιά που μου τα έστειλε.

Πρόβλημα 1: Βρείτε όλους τους πρώτους αριθμούς a,b,c και όλους τους θετικούς ακεραίους k που ικανοποιούν την εξίσωση \displaystyle{ a^2+b^2+16c^2 = 9k^2+1.}

Πρόβλημα 2: Θεωρούμε τους θετικούς πραγματικούς a,b,c που είναι τέτοιοι ώστε a+b+c = 3. Βρείτε την ελάχιστη τιμή της παράστασης

\displaystyle{ A = \frac{2-a^3}{a} +  \frac{2-b^3}{b} +  \frac{2-c^3}{c}}

Πρόβλημα 3: Δίνεται οξυγώνιο τρίγωνο ABC. Οι ευθείες \ell_1 και \ell_2 είναι κάθετες στην AB στα σημεία A και B αντίστοιχα. Οι κάθετες ευθείες από το μέσον M του AB προς τις πλευρές AC και BC του τριγώνου τέμνουν τις ευθείες \ell_1 και \ell_2 στα σημεία E και F αντίστοιχα. Αν D είναι το σημείο τομής των ευθειών EF και MC, να αποδείξετε ότι \displaystyle{ \angle ADB = \angle EMF.}

Πρόβλημα 4: Κάθε ένα από τα ακόλουθα τέσσερα σχήματα αποτελείται από τρία μοναδιαία τετράγωνα και καλείται L-σχήμα.
JBMO2015-L-shapes.png
JBMO2015-L-shapes.png (13.59 KiB) Προβλήθηκε 4264 φορές
Δίνεται ένας 5 \times 5 πίνακας αποτελούμενος από 25 μοναδιαία τετράγωνα, ένας θετικός ακέραιος k \leqslant 25 και απεριόριστος αριθμός L-σχημάτων οποιουδήποτε τύπου. Δυο παίκτες, ο A και ο B παίζουν το ακόλουθο παιγνίδι:

Ξεκινώντας με τον A, σημειώνουν εναλλάξ σε κάθε κίνησή τους ένα τετράγωνο που δεν είναι ήδη σημειωμένο, μέχρι να σημειώσουν συνολικά k μοναδιαία τετράγωνα. Μια τοποθέτηση L-σχημάτων λέγεται «καλή» αν τα L-σχήματα δεν επικαλύπτονται και καθένα από αυτά καλύπτει ακριβώς τρία μοναδιαία τετράγωνα του πίνακα που δεν είναι σημειωμένα. Ο B κερδίζει αν μετά από οποιαδήποτε καλή τοποθέτηση L-σχημάτων μένουν ακάλυπτα τουλάχιστον τρία μοναδιαία τετράγωνα που δεν είναι σημειωμένα.

Προσδιορίστε την ελάχιστη τιμή του k για την οποία ο B έχει στρατηγική νίκης.
τελευταία επεξεργασία από Demetres σε Παρ Ιουν 26, 2015 4:14 pm, έχει επεξεργασθεί 1 φορά συνολικά.


socrates
Επιμελητής
Δημοσιεύσεις: 6461
Εγγραφή: Δευ Μαρ 09, 2009 1:47 pm
Τοποθεσία: Θεσσαλονίκη
Επικοινωνία:

Re: ΘΕΜΑΤΑ JBMO 2015

#2

Μη αναγνωσμένη δημοσίευση από socrates » Παρ Ιουν 26, 2015 4:10 pm

Demetres έγραψε: Πρόβλημα 2: Θεωρούμε τους θετικούς πραγματικούς a,b,c που είναι τέτοιοι ώστε a+b+c = 3. Βρείτε την ελάχιστη τιμή της παράστασης

\displaystyle{ A = \frac{2-a^3}{a} +  \frac{2-b^3}{b} +  \frac{2-c^3}{c}}

Θα δείξουμε ότι A\geq 3 με ισότητα αν-ν a=b=c=1.

Θέτουμε ab+bc+ca=x, \ abc=y και κάνοντας πράξεις φτάνουμε στην \displaystyle{x(y+1)\geq 6y.}

Όμως, x^2\geq 3abc(a+b+c)=9y και y+1\geq 2\sqrt{y} οπότε \displaystyle{x(y+1)\geq 3\sqrt{y}\cdot 2\sqrt{y}=6y.}


Θανάσης Κοντογεώργης
gavrilos
Δημοσιεύσεις: 1032
Εγγραφή: Παρ Δεκ 07, 2012 4:11 pm

Re: ΘΕΜΑΤΑ JBMO 2015

#3

Μη αναγνωσμένη δημοσίευση από gavrilos » Παρ Ιουν 26, 2015 4:14 pm

Demetres έγραψε:Πρόβλημα 1: Βρείτε όλους τους πρώτους αριθμούς a,b,c και όλους τους θετικούς ακεραίους k που ικανοποιούν την εξίσωση \displaystyle{ a^2+b^2+16c^2 = 9k^2+1.}
Βρίσκουμε \displaystyle{a^{2}+b^{2}+c^{2}\equiv 1\pmod 3}.Δεδομένου ότι \displaystyle{a^2,b^2,c^2\equiv 0,1\pmod 3},ακριβώς δύο από τους \displaystyle{a^2,b^2,c^2} θα διαιρούνται με το \displaystyle{3}.

Λόγω της συμμετρίας που υπάρχει ως προς τους \displaystyle{a,b} στην αρχική εξίσωση,μπορούμε να θεωρήσουμε \displaystyle{2} περιπτώσεις.

\bullet Έστω ότι \displaystyle{3\mid a^2,b^2}.Λόγω του ότι το \displaystyle{3} είναι πρώτος,θα ισχύει \displaystyle{3\mid a,b}.

Επειδή οι \displaystyle{a,b} είναι πρώτοι,ισχύει αναγκαστικά \displaystyle{a=b=3}.Επομένως η αρχική γράφεται \displaystyle{16c^{2}=9k^2-17}.

Ισοδύναμα \displaystyle{(3k-4c)(3k+4c)=17}.Επίσης ισχύει \displaystyle{3k+4c>0} άρα αναγκαστικά \displaystyle{3k-4c>0}.

Επομένως η μοναδική περίπτωση είναι να ισχύουν \displastyle{3k+4c=17} και \displaystyle{3k-4c=1}.Το σύστημα αυτό έχει λύση την \displaystyle{(k,c)=(3,2)} που είναι δεκτή.

Συνεπώς μια τετράδα που ικανοποιεί είναι η \displaystyle{(a,b,c,k)=(3,3,2,3)}.

\bullet Έστω ότι \displaystyle{3\mid a^2,c^2}.Τότε ισχύει \displaystyle{3\mid a,c} κι επειδή οι \displaystyle{a,c} είναι πρώτοι συμπεραίνουμε ότι \displaystyle{a=c=3}.

Τότε η αρχική γράφεται \displaystyle{b^2+152=9k^2\Leftrightarrow \Leftrightarrow (3k-b)(3k+b)=152=8\cdot 19}.

Επειδή \displaystyle{3k+b>3k-b} κι επειδή οι δύο αυτοί όροι είναι αναγκαστικά και οι δύο άρτιοι,υπάρχουν οι εξής περιπτώσεις:

\dislplaystyle{\begin{cases}  
3k-b=2\\ 
3k+b=76 
\end{cases}\Leftrightarrow (k,b)=(13,37)\Leftrightarrow (a,b,c,k)=(3,37,3,13)} που είναι δεκτή.

\displaystyle{\begin{cases} 
3k-b=4\\ 
3k+b=38 
\end{cases}\Leftrightarrow (k,b)=(7,17)\Leftrightarrow (a,b,c,k)=(3,17,3,7)} που είναι επίσης δεκτή.

Συνοψίζοντας,οι τετράδες που ικανοποιούν είναι οι \displaystyle{(a,b,c,k)=(3,3,2,3),(3,37,3,13),(37,3,3,13),(3,17,3,7),(17,3,3,7)}.

\rule{430pt}{1pt}

Οι όροι \displaystyle{3k+b,3k-b} έχουν την ίδια αρτιοπεριττότητα.Είναι αδύνατο να είναι κι οι δύο περιττοί άρα είναι κι οι δύο άρτιοι.


Γιώργος Γαβριλόπουλος
gavrilos
Δημοσιεύσεις: 1032
Εγγραφή: Παρ Δεκ 07, 2012 4:11 pm

Re: ΘΕΜΑΤΑ JBMO 2015

#4

Μη αναγνωσμένη δημοσίευση από gavrilos » Παρ Ιουν 26, 2015 5:17 pm

Demetres έγραψε:Πρόβλημα 3: Δίνεται οξυγώνιο τρίγωνο ABC. Οι ευθείες \ell_1 και \ell_2 είναι κάθετες στην AB στα σημεία A και B αντίστοιχα. Οι κάθετες ευθείες από το μέσον M του AB προς τις πλευρές AC και BC του τριγώνου τέμνουν τις ευθείες \ell_1 και \ell_2 στα σημεία E και F αντίστοιχα. Αν D είναι το σημείο τομής των ευθειών EF και MC, να αποδείξετε ότι \displaystyle{ \angle ADB = \angle EMF.}
Έστω \displaystyle{AG\perp BC} και \displaystyle{BH\perp CA} με \displaystyle{G\in l_{2}} και \displaystyle{H\in l_{1}}.
Γεωμετρια mathematica_148(1).PNG
Γεωμετρια mathematica_148(1).PNG (15.03 KiB) Προβλήθηκε 4194 φορές
Θα δείξουμε αρχικά ότι \displaystyle{CM\perp EF}.Θα χρησιμοποιήσουμε διανύσματα.Ως γνωστόν \displaystyle{\vec{CM}=\frac{\vec{CA}+\vec{CB}}{2}}.

Επίσης \displaystyle{\vec{EF}=\vec{MF}-\vec{ME}}.Συνεπώς \displaystyle{2\vec{CM}\cdot \vec{EF}=(\vec{CA}+\vec{CB})(\vec{MF}-\vec{ME})=\vec{CA}\cdot \vec{MF}-\vec{CA}\cdot \vec{ME}+\vec{CB}\cdot \vec{MF}-\vec{CB}\cdot \vec{ME}}.

Ισχύει \displaystyle{\vec{CA}\cdot \vec{ME}=\vec{CB}\cdot \vec{MF}=0} λόγω των καθετοτήτων.Άρα \displaystyle{2\vec{CM}\cdot \vec{EF}=\vec{CA}\cdot \vec{MF}-\vec{CB}\cdot \vec{ME}}.

Επειδή \displaystyle{\vec{MF}=\frac{1}{2}\vec{AG}} θα ισχύει \displaystyle{\vec{CA}\cdot \vec{MF}=\frac{1}{2}\vec{CA}\cdot \vec{AG}=\frac{(CA)(AG)\cos (90+\hat{C})}{2}\overset{(AG)=\frac{(AB)}{\sin \hat{B}}}=-\frac{(CA)(AB)\sin \hat{C}}{2\sin \hat{B}}=-\frac{(AB)^{2}}{2}}

όπου στην τελευταία χρησιμοποιήθηκε ο νόμος των ημιτόνων.

Επίσης \displaystyle{\vec{CB}\cdot \vec{ME}=\frac{1}{2}\vec{CB}\cdot \vec{BH}=\frac{(CB)(BH)\cos (90+\hat{C})}{2}\overset{BH=\frac{(AB)}{\sin \hat{A}}}=-\frac{(CB)(AB)\sin \hat{C}}{2\sin \hat{A}}=-\frac{(AB)^2}{2}}

όπου στην τελευταία ισότητα χρησιμοποιήθηκε ο νόμος ημιτόνων.Συμπεραίνουμε ότι \displaystyle{2\vec{CM}\cdot \vec{EF}=-\frac{(AB)^2}{2}+\frac{(AB)^2}{2}=0\Leftrightarrow \vec{CM}\perp \vec{EF}} και τέλος.

\rule{430pt}{1pt}
Γεωμετρια mathematica_148(2).PNG
Γεωμετρια mathematica_148(2).PNG (14.8 KiB) Προβλήθηκε 4194 φορές
Πίσω στο αρχικό πρόβλημα,ισχύουν \displaystyle{\angle{MDF}=\angle{MBF}} άρα το \displaystyle{MDFB} είναι εγγράψιμο.

Επίσης \displaystyle{\angle{MDE}=\angle{MAE}} άρα και το \displaystyle{MAED} είναι εγγράψιμο.

Επομένως \displaystyle{\angle{MAD}=\angle{MEF}} και \displaystyle{\angle{MFE}=\angle{DBA}} άρα \displaystyle{\triangle{ADB}\simeq \triangle{EMF}} και το ζητούμενο αποδείχθηκε.


Γιώργος Γαβριλόπουλος
Άβαταρ μέλους
Κώστας Παππέλης
Δημοσιεύσεις: 261
Εγγραφή: Παρ Ιούλ 24, 2009 4:17 pm
Τοποθεσία: Αθήνα

Re: ΘΕΜΑΤΑ JBMO 2015

#5

Μη αναγνωσμένη δημοσίευση από Κώστας Παππέλης » Παρ Ιουν 26, 2015 6:22 pm

Εναλλακτικά για το 3,

εάν S είναι το σημείο τομής των ME, AC και T το κοινό σημείο των MF, BC έχουμε MF*MT=MB^2=MA^2=MS*ME δηλαδή το TFES είναι εγγράψιμο. Άρα χρησιμοποιώντας και το εγγράψιμο MSCT είναι: <MFE=<MST=<MCT άρα το TDCM είναι επίσης εγγράψιμο, οπότε η γωνία <FDC=90, από το οποίο έπεται εύκολα το ζητούμενο όπως στη λύση παραπάνω του gavrilos.

Δύσκολα τα θέματα για JBMO, αλλά πολύ ωραία.


gavrilos
Δημοσιεύσεις: 1032
Εγγραφή: Παρ Δεκ 07, 2012 4:11 pm

Re: ΘΕΜΑΤΑ JBMO 2015

#6

Μη αναγνωσμένη δημοσίευση από gavrilos » Σάβ Ιουν 27, 2015 11:28 pm

Demetres έγραψε:Δίνεται ένας 5 \times 5 πίνακας αποτελούμενος από 25 μοναδιαία τετράγωνα, ένας θετικός ακέραιος k \leqslant 25 και απεριόριστος αριθμός L-σχημάτων οποιουδήποτε τύπου. Δυο παίκτες, ο A και ο B παίζουν το ακόλουθο παιγνίδι:

Ξεκινώντας με τον A, σημειώνουν εναλλάξ σε κάθε κίνησή τους ένα τετράγωνο που δεν είναι ήδη σημειωμένο, μέχρι να σημειώσουν συνολικά k μοναδιαία τετράγωνα. Μια τοποθέτηση L-σχημάτων λέγεται «καλή» αν τα L-σχήματα δεν επικαλύπτονται και καθένα από αυτά καλύπτει ακριβώς τρία μοναδιαία τετράγωνα του πίνακα που δεν είναι σημειωμένα. Ο B κερδίζει αν μετά από οποιαδήποτε καλή τοποθέτηση L-σχημάτων μένουν ακάλυπτα τουλάχιστον τρία μοναδιαία τετράγωνα που δεν είναι σημειωμένα.

Προσδιορίστε την ελάχιστη τιμή του k για την οποία ο B έχει στρατηγική νίκης.
Κατ' αρχάς συγχαρητήρια σε όλα τα παιδιά από την Ελλάδα που συμμετείχαν στο διαγωνισμό!Είχαμε για άλλη μια χρονιά πολύ καλή εμφάνιση!

Θα προσπαθήσω να προσεγγίσω και το τελευταίο πρόβλημα.

Θα ήθελα να δω και τη λύση που έχει υπ' όψιν του και ο Δημήτρης που το πρότεινε (μπράβο,και επί τη ευκαιρία,για το πρόβλημα!).
Συνδυαστική JBMO 2015.PNG
Συνδυαστική JBMO 2015.PNG (232.54 KiB) Προβλήθηκε 3821 φορές
Θα δείξουμε αρχικά πως για \displaystyle{k=4} ο \displaystyle{B} έχει στρατηγική νίκης.

\bullet Αν ο \displaystyle{A} σημειώσει στην πρώτη κίνηση κάποιο από τα τετράγωνα από τα οποία περνά κάποια κόκκινη γραμμή,τότε ο \displaystyle{B} παίζει έτσι ώστε στο τέλος των \displaystyle{4} κινήσεων να υπάρχει κάποια γραμμή ή στήλη από αυτές που αντιστοιχούν σε κόκκινες γραμμές,στην οποία να υπάρχουν \displaystyle{3} σημειωμένα τετράγωνα.Για παράδειγμα αν ο \displaystyle{A} σημειώσει το \displaystyle{A2} τότε ο \displaystyle{B} θέλει στο τέλος των κινήσεων να είναι σημειωμένα τα \displaystyle{B2,C2}.Αυτό προφανώς μπορεί να γίνει αφού ο \displaystyle{B} έχει δύο κινήσεις.

Στην περίπτωση αυτή,είναι εμφανές ότι τα \displaystyle{A1,B1} δεν μπορούν να καλυφθούν σε κάποια καλή τοποθέτηση.Ακόμη κι αν ο \displaystyle{A} στην δεύτερη κίνησή του καλύψει ένα από αυτά,τότε μένει ένα ακάλυπτο.Δεδομένου ότι σε κάθε καλή τοποθέτηση το πλήθος των καλυμμένων τετραγώνων είναι πολλαπλάσιο του \displaystyle{3},και πως μετά τις \displaystyle{4} κινήσεις μένουν \displaystyle{21} μη σημειωμένα τετράγωνα,τότε αφού \displaystyle{1} από αυτά θα μείνει οπωσδήποτε ακάλυπτο,θα καλυφθούν το πολύ \displaystyle{18} τετράγωνα και ο \displaystyle{B} θα κερδίσει (ομοίως,αν π.χ. ο \displaystyle{A} σημειώσει το \displaystyle{D2} τότε ο \displaystyle{B} φροντίζει ώστε να σημειωθούν τα \displaystyle{D1,D3} ή τα \displaystyle{C2,E2} και τότε θα έχει κερδίσει όπως περιγράψαμε παραπάνω).

\bullet Αν ο \displaystyle{A} σημειώσει στην αρχή ένα γωνιακό τετράγωνο τότε ο \displaystyle{B} σημειώνει το τετράγωνο που μοιράζεται μόνο μία κορυφή με αυτό που σημείωσε ο \displaystyle{A} (αν π.χ. ο \displaystyle{A} σημειώσει το \displaystyle{A1} τότε ο \displaystyle{B} σημειώνει το \displaystyle{B2}).Όποια και να είναι η επόμενη κίνηση του \displaystyle{A} ο \displaystyle{B} μπορεί να "εγκλωβίσει" ένα από τα γειτονικά τετράγωνα του γωνιακού (στο προηγούμενο παράδειγμα δηλαδή,ένα εκ των \displaystyle{A2,B1}).Αυτό μπορεί να το κάνει ως εξής:αν ο \displaystyle{A} στη δεύτερη κίνηση δε σημειώσει το \displaystyle{B1} τότε ο \displaystyle{B} σημειώνει το \displaystyle{C1} κι έτσι "εγκλωβίζει" το \displaystyle{B1},αλλιώς σημειώνει το \displaystyle{A3} και "εγκλωβίζει" το \displaystyle{A2}.Λόγω του επιχειρήματος με τα πολλαπλάσια του \displaystyle{3} θα κερδίσει και πάλι.

\bullet Αν ο \displaystyle{A} σημειώσει στην αρχή κάποιο εκ των \displaystyle{C1,C5,A3,E3} τότε ο \displaystyle{B} μπορεί να φροντίσει ώστε να σημειωθούν αντίστοιχα τα ζεύγη \displaystyle{(D2,E2),(D4,E4),(B1,B2),(D4,D5)} και τότε θα έχουν "εγκλωβιστεί" σε κάθε περίπτωση δύο τετράγωνα που βρίσκονται στην περιφέρεια του πίνακα.Ακόμη κι αν ο \displaystyle{A} σημειώσει στη δεύτερη κίνηση κάποιο από αυτά,τότε μένει ένα που δεν μπορεί να καλυφθεί από καλή τοποθέτηση,και με βάση το επιχείρημα με τα πολλαπλάσια του \displaystyle{3} ο \displaystyle{B} κερδίζει και πάλι.

\bullet Αν ο \displaystyle{A} σημειώσει στην πρώτη κίνηση το \displaystyle{C3},τότε ο \displaystyle{B} σημειώνει το \displaystyle{B1}.Αν ο \displaystyle{A} δε σημειώσει στη δεύτερη κίνησή του το \displaystyle{A1} τότε ο \displaystyle{B} σημειώνει το \displaystyle{A2} και το \displaystyle{A1} "εγκλωβίζεται" όποτε ο \displaystyle{B} κερδίζει όπως και στις προηγούμενες περιπτώσεις.Έστω ότι ο \displaystyle{A} σημειώνει και το \displaystyle{A1}\displaystyle{B} μετά μπορεί να σημειώσει το \displaystyle{C1}.Μετά,υπάρχουν \displaystyle{3} τρόποι να καλυφθεί το \displaystyle{C2} σε μια καλή τοποθέτηση.Ο πρώτος είναι με το \displaystyle{L} που καλύπτει τα \displaystyle{C2,D1,D2}.Τότε το \displaystyle{E1} δεν μπορεί να καλυφθεί και ο \displaystyle{B} κερδίζει.Ο άλλος τρόπος είναι να χρησιμοποιηθεί το \displaystyle{L} που καλύπτει τα \displaystyle{C2,D2,D3}.Τότε αναγκαστικά θα χρησιμοποιηθεί και το \displaystyle{L} που καλύπτει τα \displaystyle{D1,E1,E2}.Τώρα πια για να καλυφθεί το \displaystyle{E3} πρέπει να χρησιμοποιηθεί το \displaystyle{L} που καλύπτει τα \displaystyle{E3,E4,D4}.Όμως τότε το \displaystyle{E5} δεν μπορεί να καλυφθεί άρα ο \displaystyle{B} κερδίζει.Τέλος,μπορεί να χρησιμοποιηθεί το \displaystyle{L} που καλύπτει τα \displaystyle{C2,B2,B3}.Τότε όμως δεν μπορεί να καλυφθεί το \displaystyle{A2} οπότε ο \displaystyle{B} και πάλι κερδίζει.

Επομένως σε κάθε περίπτωση ο \displaystyle{B} μπορεί να κερδίσει αν \displaystyle{k=4}.

Μένει να δείξουμε πως ο \displaystyle{B} δεν έχει την τύχη στα χέρια του αν \displaystyle{k\leq 3}.Για \displaystyle{k=1} αυτό είναι προφανές.Θα εξετάσουμε τώρα την περίπτωση \displaystyle{k=2}.Υποθέτουμε ότι αρχικά ο \displaystyle{A} σημειώνει το κεντρικό τετράγωνο.Λόγω συμμετρίας υπάρχουν \displaystyle{3} περιπτώσεις:

1. Ο \displaystyle{B} επιλέγει ένα εκ των \displaystyle{A1,B2}.

2. Ο \displaystyle{B} επιλέγει το \displaystyle{B1}.

3. Ο \displaystyle{B} επιλέγει ένα εκ των \displaystyle{C2,C1}.

Όλες οι άλλες περιπτώσεις ανάγονται σε αυτές τις \displaystyle{3}.Για κάθε μια εύκολα βλέπουμε πως υπάρχει καλή τοποθέτηση ώστε να κερδίζει ο \displaystyle{A} (για να μην εξετάζετε πολλές περιπτώσεις ψάξτε π.χ. για καλή τοποθέτηση που αφήνει ακάλυπτα τα \displaystyle{A1,B2} μαζί ώστε να καλύψετε μαζί τις περιπτώσεις όπου ο \displaystyle{B} επιλέγει κάποιο από αυτά).

Η περίπτωση \displaystyle{k=3} απορρίπτεται εύκολα.Στην προηγούμενη περίπτωση ο,τι και να επιλέξει ο \displaystyle{B} υπάρχει καλή τοποθέτηση ώστε να κερδίζει ο \displaystyle{A}.Όμως σε κάθε τέτοια καλή τοποθέτηση μένουν \displaystyle{2} τετράγωνα κενά.Ο \displaystyle{A} λοιπόν μετά την κίνηση του \displaystyle{B} έχει δύο (τουλάχιστον) πιθανές επιλογές ώστε να κερδίσει.


Γιώργος Γαβριλόπουλος
Άβαταρ μέλους
Demetres
Γενικός Συντονιστής
Δημοσιεύσεις: 8989
Εγγραφή: Δευ Ιαν 19, 2009 5:16 pm
Τοποθεσία: Λεμεσός/Πύλα
Επικοινωνία:

Re: ΘΕΜΑΤΑ JBMO 2015

#7

Μη αναγνωσμένη δημοσίευση από Demetres » Κυρ Ιουν 28, 2015 12:16 am

Βάζω και την προτεινόμενη λύση για το 4.

Αν k=1, ο A σημειώνει το άνω αριστερά τετραγωνάκι και ακολούθως καλύπτει την σκακιέρα ως εξής:
JBMO2015Fig1-small.png
JBMO2015Fig1-small.png (6.69 KiB) Προβλήθηκε 3790 φορές
Αν k=2, ο A σημειώνει το άνω αριστερά τετραγωνάκι. Όποιο τετραγωνάκι και να σημειώσει ο B, ο A συμπληρώνει την σκακιέρα ακριβώς όπως προηγουμένως με την εξαίρεση ότι δεν τοποθετεί το L-σχήμα που καλύπτει το σημαδεμένο τετράγωνο του B. O A κερδίζει αφού έμειναν μόνο δύο ακάλυπτα τετράγωνα. Για k=3 ο A κερδίζει με ακριβώς την ίδια στρατηγική. Σαν δεύτερο τετράγωνο σημαδεύει οποιοδήποτε από τα δύο άδεια τετράγωνα του L-σχήματος που καλύπτει το τετράγωνo του B.

Ας δείξουμε τώρα ότι για k=4 κερδίζει ο B. Αφού θα μείνουν 21 ασημείωτα τετράγωνα, ο A πρέπει να τα καλύψει όλα με 7 L-σχήματα.

Μπορούμε να υποθέσουμε ότι στην πρώτη του κίνηση ο A δεν καλύπτει κανένα τετράγωνο στις τελευταίες δύο γραμμές τις σκακιέρας. (Αλλιώς την περιστρέφουμε.) Στην πρώτη του κίνηση ο B σημειώνει το τετράγωνο 1 της πιο κάτω εικόνας.
JBMO2015Fig3-small.png
JBMO2015Fig3-small.png (4.23 KiB) Προβλήθηκε 3790 φορές
Αν ο A στην επόμενή του κίνηση δεν σημειώσει κάποιο από τα 2,3 and 4 ο B σημειώνει το 3. Τότε ο B κερδίζει αφού το 2 μένει ασημείωτο και δεν μπορεί να καλυφθεί με L-σχήματα.

Αν ο A σημειώσει το 2 ο B σημειώνει το 5. Τότε ο B κερδίζει αφού το 3 μένει ασημείωτο και δεν μπορεί να καλυφθεί με L-σχήματα.

Τέλος αν ο A σημειώσει το 3 ή το 4 τότε ο B σημειώνει οποιοδήποτε άσπρο τετράγωνο στο πιο κάτω σχήμα.
JBMO2015Fig2-small.png
JBMO2015Fig2-small.png (2.44 KiB) Προβλήθηκε 3790 φορές
Παρατηρήστε ότι υπάρχουν τουλάχιστον 8 ασημείωτα μαύρα τετράγωνα. Κάθε L-σχήμα καλύπτει το πολύ ένα από αυτά. Αλλά ο A μπορεί να χρησιμοποιήσει μόνο 7 L-σχήματα, και άρα πάλι κάποιο θα μείνει ακάλυπτο οπότε θα κερδίσει ο B

Έχουμε καλύψει όλες τις περιπτώσεις οπότε ο B κερδίζει όταν k=4.


panagiotis99
Δημοσιεύσεις: 133
Εγγραφή: Δευ Φεβ 04, 2013 8:24 pm
Τοποθεσία: Αθηνα

Re: ΘΕΜΑΤΑ JBMO 2015

#8

Μη αναγνωσμένη δημοσίευση από panagiotis99 » Κυρ Ιουν 28, 2015 12:33 pm

Καλησπέρα :logo: βάζω μια προσέγγιση ελπίζω διαφορετική απο τις προηγούμενες.
Δουλεύω στο σχήμα που έφτιαξε ο Γιώργος (gavrilos).

Aρχικά σκοπός του B είναι να σχηματίσει δυάδες η τριάδες που θα μπλόκαρουν ένα η δύο τετραγωνάκια αντίστοιχα και έτσι θα κερδίσει.
Πιο αναλυτικά έχουμε 3 περιπτώσεις

Α) Ο A σημαδεύει ένα τετραγωνάκι που βρίσκεται στην γραμμή 1 ή 5ή στην στήλη A ή E. Tότε οBσημειώνει ένα τετραγωνάκι που βρίσκεται διαγωνια και στην επομενή του κίνηση σημειώνει ένα τετραγωνάκι που βρίσκεται οριζόντια η καθέτα του πρώτου π.χ (αν ο A σημειώσει το B1 τότε ο B σημειώνει το C2 και το D2 και μπλοκάρει το C1 άρα κερδίζει.

B) Ο A σημαδεύει ένα τετραγωνάκι που βρίσκεται στην γραμμή 2 ή 4 ή στην στήλη B ή D. Αν το τετραγωνακι δεν είναι στην στήλη C η στην γραμμή 3 τότε ο Β σημαδεύει ένα τετραγωνάκι τέτοιο ώστε να σχηματιζει δυαδα με του A οριζόντια ή κάθετα και να έιναι στην γραμμή 1 ή 5 ή στην στήλη A ή E και έτσι κερδίζει πχ (ο A το B2 o B το B1). Άρα μπλοκάρεται το A1.Αν το τετραγωνακι είναι στην στήλη C η στην γραμμή 3 τότε επιθυμεί να σχηματίσει μια κάθετη η οριζόντια τριάδα. πχ (ο A τo B3 ο B το B2 και το B1). Άρα μπλοκάρονται ταA1,A2 και κερδίζει

C)O A σημαδεύει το C3.O B σημαδεύει τα D4,D5 μπλοκάρεται το E5 και κερδίζει. Αν ο A σημαδέψει το E5 τότε ο B σημαδεύει το C5 και μπλοκάρεται το D4 άρα παλι κερδιζει


panagiotis99
Δημοσιεύσεις: 133
Εγγραφή: Δευ Φεβ 04, 2013 8:24 pm
Τοποθεσία: Αθηνα

Re: ΘΕΜΑΤΑ JBMO 2015

#9

Μη αναγνωσμένη δημοσίευση από panagiotis99 » Κυρ Ιουν 28, 2015 4:58 pm

Βάζω και την προσέγγιση μου για την ανισότητα.
Θα δείξω ότι ελάχιστη τιμή είναι το 3 με ισότητα στο (a,b,c)-->(1,1,1)
Η δοθείσα γράφεται:
2\sum{\frac{1}{a}}+\sum{-a^2}\geq 3 \Rightarrow 2\sum{\frac{1}{a}}+2\sum{ab}\geq 3 +(a+b+c)^2 \Rightarrow \sum{\frac{1}{a}}+\sum{ab}\geq 6

Χρησιμοποιοώντας την ανισότητα ΑΜ-ΓΜ έχουμε

\sum{\frac{1}{a}}+\sum{ab}\geq 2\sqrt{\sum{\frac{1}{a}}\sum{ab}}=2\sqrt{2(a+b+c) + \frac{ab}{c}+\frac{bc}{a}+\frac{ca}{b}}=2\sqrt{6+ \frac{ab}{c}+\frac{bc}  {a}+\frac{ca}{b}}= 2\sqrt{6+\sum({\frac{ab}{2c} +\frac{bc}{2a}})}  \geq 2\sqrt{6+\sum{b}}=6

q.e.d


Αλέξανδρος.Θ
Δημοσιεύσεις: 11
Εγγραφή: Σάβ Νοέμ 21, 2015 5:21 pm

Re: ΘΕΜΑΤΑ JBMO 2015

#10

Μη αναγνωσμένη δημοσίευση από Αλέξανδρος.Θ » Τρί Νοέμ 24, 2015 6:13 pm

Ως αναφορά το πρώτο θέμα . Μπορεί κάποιος να το εξηγήσει πιο αναλυτικά γιατι δεν το κατάλαβα? Πώς μπορώ να καταλήξω στο a^2 + b^2+ c^2 = 1 (mod 3)


Άβαταρ μέλους
Demetres
Γενικός Συντονιστής
Δημοσιεύσεις: 8989
Εγγραφή: Δευ Ιαν 19, 2009 5:16 pm
Τοποθεσία: Λεμεσός/Πύλα
Επικοινωνία:

Re: ΘΕΜΑΤΑ JBMO 2015

#11

Μη αναγνωσμένη δημοσίευση από Demetres » Τρί Νοέμ 24, 2015 6:24 pm

Αλέξανδρε, καλωσόρισες.

Η φράση m = n \bmod 3 σημαίνει ότι το m-n είναι πολλαπλάσιο του 3. Π.χ. 5 \equiv 17 \bmod 3 αλλά 8 \not \equiv 12 \bmod 3.

Από την αρχική εξίσωση έχουμε a^2 + b^2 + c^2 - 1 = 9k^2-15c^2 = 3(3k^2-5c^2) οπότε το a^2+b^2+c^2 - 1 είναι πολλαπλάσιο του 3.

Ο τρόπος που σκεφτόμαστε συνήθως είναι ότι και από τα δύο μέρη μέρη της ισότητας a^2+b^2+16c^2 = 9k^2 + 1 μπορώ να αφαιρέσω κάποιο πολλαπλάσιο του 3 αρκεί βέβαια στο τέλος να γράψω ότι η ισότητα είναι \bmod 3. Π.χ. αφαιρώντας 15c^2 από το αριστερό και 9k^2 από το δεξί μπορώ να γράψω απευθείας a^2+b^2+c^2 \equiv 1 \bmod 3.

Ελπίζω τα πιο πάνω να βοήθησαν. Προσπάθησε να διαβάσεις και την υπόλοιπη λύση και πες μας αν κολλήσεις κάπου. Για το δεύτερο βήμα προσπάθησε να δείξεις ότι n \equiv 0 \bmod 3 ή n \equiv 1 \bmod 3 οτιδήποτε και αν είναι το n.


Αλέξανδρος.Θ
Δημοσιεύσεις: 11
Εγγραφή: Σάβ Νοέμ 21, 2015 5:21 pm

Re: ΘΕΜΑΤΑ JBMO 2015

#12

Μη αναγνωσμένη δημοσίευση από Αλέξανδρος.Θ » Τρί Νοέμ 24, 2015 6:45 pm

Ευχαριστώ πολύ.


Άβαταρ μέλους
Ανδρέας Πούλος
Δημοσιεύσεις: 1494
Εγγραφή: Κυρ Μαρ 01, 2009 10:47 pm
Τοποθεσία: ΘΕΣΣΑΛΟΝΙΚΗ
Επικοινωνία:

Re: ΘΕΜΑΤΑ JBMO 2015

#13

Μη αναγνωσμένη δημοσίευση από Ανδρέας Πούλος » Τετ Νοέμ 25, 2015 2:31 am

Το θέμα της Γεωμετρίας με ένα γρήγορο κοίταγμα πρέπει να προκύπτει και με Αναλυτική Γεωμετρία.
Ορίζουμε M(0, 0), A(0, 1), B(0, -1), C(a, b).
Εκφράζουμε τις συντεταγμένες των σημείων E, D, F συναρτήσει των αριθμών a, b.
Δείχνουμε ότι η γωνία των διανυσμάτων DA, DB είναι ίση με αυτή των διανυσμάτων ME, MF.
Θα το δοκιμάσω το πρωί, αν και καμιά φορά τα δύσκολα (από άποψη πράξεων) φαίνονται για εύκολα.
Προφανώς, η τεχνική του Gavrilos με τη Διανυσματική Γεωμετρία δούλεψε μια χαρά και μου άρεσε πολύ.

Ανδρέας Πούλος


Άβαταρ μέλους
Soteris
Δημοσιεύσεις: 466
Εγγραφή: Δευ Ιούλ 21, 2014 1:59 pm
Τοποθεσία: Λάρνακα, Κύπρος

Re: ΘΕΜΑΤΑ JBMO 2015

#14

Μη αναγνωσμένη δημοσίευση από Soteris » Τετ Νοέμ 25, 2015 8:16 am

Ανδρέας Πούλος έγραψε:Το θέμα της Γεωμετρίας με ένα γρήγορο κοίταγμα πρέπει να προκύπτει και με Αναλυτική Γεωμετρία.

Ανδρέας Πούλος
Καλημέρα. Μιας και επανήλθε το θέμα στο προσκήνιο, να πω για την ιστορία ότι υπήρξαν αρκετοί μαθητές που αντιμετώπισαν το πρόβλημα στα πλαίσια της Αναλυτικής Γεωμετρίας. Ένας από αυτούς ήταν και ο Άγγελος Άσσος, μέλος της κυπριακής αποστολής, ο οποίος πέτυχε αργυρό μετάλλιο. Δεν θα παραθέσω λύση για να σας αφήσω να την χαρείτε.


Σωτήρης Λοϊζιάς
Άβαταρ μέλους
silouan
Επιμελητής
Δημοσιεύσεις: 1398
Εγγραφή: Τρί Ιαν 27, 2009 10:52 pm

Re: ΘΕΜΑΤΑ JBMO 2015

#15

Μη αναγνωσμένη δημοσίευση από silouan » Τετ Νοέμ 25, 2015 2:19 pm

Στο αρχείο που επισυνάπτω έχω μαζέψει τις πιο αντιπροσωπευτικές λύσεις για τα προβλήματα (άλλες δικές μου, άλλες που άκουσα στο διαγωνισμό, άλλες όπως ήταν στο επίσημο φυλλάδιο) και κάθε πρόβλημα έχει 2-3 λύσεις. Ελπίζω να το χαρείτε.
Συνημμένα
greeksolutions.pdf
(206.25 KiB) Μεταφορτώθηκε 221 φορές


Σιλουανός Μπραζιτίκος
Άβαταρ μέλους
Soteris
Δημοσιεύσεις: 466
Εγγραφή: Δευ Ιούλ 21, 2014 1:59 pm
Τοποθεσία: Λάρνακα, Κύπρος

Re: ΘΕΜΑΤΑ JBMO 2015

#16

Μη αναγνωσμένη δημοσίευση από Soteris » Τετ Νοέμ 25, 2015 2:30 pm

smar έγραψε:Στο αρχείο που επισυνάπτω έχω μαζέψει τις πιο αντιπροσωπευτικές λύσεις για τα προβλήματα (άλλες δικές μου, άλλες που άκουσα στο διαγωνισμό, άλλες όπως ήταν στο επίσημο φυλλάδιο) και κάθε πρόβλημα έχει 2-3 λύσεις. Ελπίζω να το χαρείτε.
Πολύ χρήσιμο Σιλουανέ!!! Να 'σαι καλά φίλε..

:coolspeak:


Σωτήρης Λοϊζιάς
Άβαταρ μέλους
S.E.Louridas
Δημοσιεύσεις: 5956
Εγγραφή: Σάβ Μαρ 21, 2009 10:53 am
Τοποθεσία: Aegaleo.
Επικοινωνία:

Re: ΘΕΜΑΤΑ JBMO 2015

#17

Μη αναγνωσμένη δημοσίευση από S.E.Louridas » Τετ Νοέμ 25, 2015 8:39 pm

Μία διαπραγμάτευση για το όμορφο 3ο θέμα της Γεωμετρίας του φίλου Θεόκλητου (από την όμορφη Σάμο ... για την αδελφή Κύπρο).

Θεωρούμε το παραλληλόγραμμο VQFE κέντρου M, ώστε \vartriangle MEV = \vartriangle MFQ \sim \vartriangle CAB,\;\;\mu \varepsilon \;\angle EMV =\angle C,\;\;\angle MVE = \angle B,\;\,\angle VEM = \angle A. Τότε η ευθεία της διαμέσου M{A_1} που διέρχεται από το μέσο B_1 της QF θα είναι κάθετη στη διάμεσο MC αφού AB \bot VE. Οπότε παίρνουμε {A_1}{B_1}\parallel EF \Rightarrow MC \bot EF. Έτσι οδηγούμαστε στην εγγραψιμμότητα των τετραπλεύρων AMDE,\;MBFD που δίνει αυτόματα την ζητούμενη ισότητα.
Συνημμένα
Κύπρος.png
Κύπρος.png (14.36 KiB) Προβλήθηκε 2832 φορές


S.E.Louridas

1.Μιλώ, μόνο όταν έχω να πώ κάτι καλύτερο από την σιωπή (Πυθαγόρας).
2.Οι αξίες αντανακλώνται, Δεν επιβάλλονται.
3.Είναι Κορυφαία η κάθε στιγμή επίλυσης ενός Μαθηματικού προβλήματος.
Άβαταρ μέλους
Διονύσιος Αδαμόπουλος
Δημοσιεύσεις: 807
Εγγραφή: Σάβ Μαρ 19, 2016 5:11 pm
Τοποθεσία: Πύργος Ηλείας

Re: ΘΕΜΑΤΑ JBMO 2015

#18

Μη αναγνωσμένη δημοσίευση από Διονύσιος Αδαμόπουλος » Κυρ Δεκ 11, 2016 11:17 pm

Demetres έγραψε: Πρόβλημα 3: Δίνεται οξυγώνιο τρίγωνο ABC. Οι ευθείες \ell_1 και \ell_2 είναι κάθετες στην AB στα σημεία A και B αντίστοιχα. Οι κάθετες ευθείες από το μέσον M του AB προς τις πλευρές AC και BC του τριγώνου τέμνουν τις ευθείες \ell_1 και \ell_2 στα σημεία E και F αντίστοιχα. Αν D είναι το σημείο τομής των ευθειών EF και MC, να αποδείξετε ότι \displaystyle{ \angle ADB = \angle EMF.}
Φέρνουμε τις CE και CF.

Εφαρμόζουμε αποκλειστικά κριτήρια καθετότητας και Π.Θ:

Επειδή EM \perp AC θα ισχύει ότι: AE^2-CE^2=AM^2-CM^2 (1)

Επειδή FM \perp BC θα ισχύει ότι: BF^2-CF^2=BM^2-CM^2 \Rightarrow BF^2-CF^2=AM^2-CM^2(2)

Από τις σχέσεις (1) και (2) έχουμε: AE^2-CE^2=BF^2-CF^2  \Rightarrow AE^2-BF^2=CE^2-CF^2 (3)

Με Π.Θ. στο ορθογώνιο τρίγωνο AEM προκύπτει: AE^2=EM^2-AM^2 (4)

Με Π.Θ. στο ορθογώνιο τρίγωνο BFM προκύπτει: BF^2=FM^2-BM^2 \Rightarrow BF^2=FM^2-AM^2 (5)

Αφαιρώντας κατά μέλη τις (4) και (5) έχουμε: AE^2-BF^2=EM^2-FM^2 (6)

Από τις σχέσεις (3) και (6) προκύπτει ότι: CE^2-CF^2=EM^2-FM^2 \Rightarrow CM \perp EF

Η συνέχεια είναι η γνωστή...
Συνημμένα
JBMO 2015.png
JBMO 2015.png (32.14 KiB) Προβλήθηκε 2099 φορές


Houston, we have a problem!
Άβαταρ μέλους
Διονύσιος Αδαμόπουλος
Δημοσιεύσεις: 807
Εγγραφή: Σάβ Μαρ 19, 2016 5:11 pm
Τοποθεσία: Πύργος Ηλείας

Re: ΘΕΜΑΤΑ JBMO 2015

#19

Μη αναγνωσμένη δημοσίευση από Διονύσιος Αδαμόπουλος » Σάβ Μάιος 13, 2017 10:10 pm

Διονύσιος Αδαμόπουλος έγραψε:
Demetres έγραψε: Πρόβλημα 3: Δίνεται οξυγώνιο τρίγωνο ABC. Οι ευθείες \ell_1 και \ell_2 είναι κάθετες στην AB στα σημεία A και B αντίστοιχα. Οι κάθετες ευθείες από το μέσον M του AB προς τις πλευρές AC και BC του τριγώνου τέμνουν τις ευθείες \ell_1 και \ell_2 στα σημεία E και F αντίστοιχα. Αν D είναι το σημείο τομής των ευθειών EF και MC, να αποδείξετε ότι \displaystyle{ \angle ADB = \angle EMF.}
Εφαρμόζουμε αποκλειστικά κριτήρια καθετότητας και Π.Θ:
...
Εναλλακτικά, για να αποδείξουμε ότι EF\perp CM, με διανύσματα :

\vec{EF} \cdot \vec{CM} = (\vec{EM}+\vec{MF}) \cdot \dfrac{\vec{CA}+\vec{CB}}{2} =

=\dfrac{1}{2}\left(\vec{EM} \cdot \vec{CA} + \vec{EM} \cdot \vec{CB} + \vec{MF} \cdot \vec{CA} + \vec{MF} \cdot \vec{CB}\right) = \dfrac{1}{2}\left(0 + \vec{EM} \cdot \vec{CB} + \vec{MF} \cdot \vec{CA} + 0\right) =

=\dfrac{1}{2}\left(\vec{EM} \cdot (\vec{CA}-\vec{BA}) + \vec{MF} \cdot ( \vec{CB}-\vec{AB}) \right) = \dfrac{1}{2}\left(\vec{EM} \cdot \vec{CA}- \vec{EM} \cdot \vec{BA} + \vec{MF} \cdot  \vec{CB}- \vec{MF} \cdot \vec{AB}\right) =

=\dfrac{1}{2}\left(0 - \vec{EM} \cdot \vec{BA} + 0 - \vec{MF} \cdot \vec{AB}\right) = \dfrac{1}{2}\left(\vec{EM} \cdot \vec{AB} + \vec{FM} \cdot \vec{AB}\right) =

=\dfrac{1}{2}\left(\pi\rho o \beta _{\vec{AB}}\vec{EM} \cdot \vec{AB} + \pi\rho o \beta _{\vec{AB}}\vec{FM}\cdot \vec{AB}\right) = \dfrac{1}{2}\left(\vec{AM} \cdot \vec{AB} + \vec{BM} \cdot \vec{AB}\right) =

=\dfrac{1}{2}\left(\vec{AM} - \vec{MB} \right)\cdot \vec{AB} = 0

Άρα EF\perp CM

Η συνέχεια είναι η γνωστή...


Houston, we have a problem!
Απάντηση

Επιστροφή σε “Θέματα διαγωνισμών (ΕΜΕ, ΚΥΜΕ, BMO, JBMO, IMO, Kangaroo κλπ)”

Μέλη σε σύνδεση

Μέλη σε αυτήν τη Δ. Συζήτηση: Δεν υπάρχουν εγγεγραμμένα μέλη και 4 επισκέπτες